Jump to content
  • Member Statistics

    17,528
    Total Members
    7,904
    Most Online
    Gonzalo00
    Newest Member
    Gonzalo00
    Joined

The Post Christmas storm threat is still vey real


Damage In Tolland

Recommended Posts

Verbatim, it looks as though I would be having Blizzard conditions at 84h per the GFS.

Station ID: KTAN Lat: 41.87 Long: 71.02

GFS Model Run: 0Z 24DEC 2010

HR Valid 2m Tmp 2m Dpt 10m Dir 10m Spd TPrcp CPrcp 1000-500 500mb 850mb 500mb MSLP TCC PRS WX Low Middle High Max Min Sfc

Deg F Deg F deg kt in. in. Thk GPH Tmp Tmp mb % TEXT Clouds Clouds Clouds Tmp Tmp Vis

84 12/27 12Z 29 28 350 32 0.63 0.00 530 515 -6.6 -23.9 980.5 100 SN 000OVC107 107OVC192 218BKN271 32 29 0.0

I don't need to look at FOUS to know that; I suspect that a good portion of E MA is experiencing blizz. conditions at some point.

Link to comment
Share on other sites

  • Replies 1.8k
  • Created
  • Last Reply

1031-964mb gradient on the 00z GFS

Regardless of how much snow falls, I'm not sure folks understand the ramifications of that; holy sh**.

3 mb more potent than Feb 1978...3.5 day range......that's cute at day 5-7, but it's sobering within day 4.

Put some boulders in the garbage cans....

Link to comment
Share on other sites

The operational GFS can do whatever it wants. :)

Keep in mind the GEFS are run at a much lower resolution than the operational models. I'd wait for the EC/EC ens before believing the op GFS.

Yes and that's why I posted those maps. I think sometimes the GFS Ens are given more credit than they deserve and I'm hoping this is one of those times.

Link to comment
Share on other sites

Identical to 12Z

A bit too far east for my liking.

Is it the same or east? You're confusing me, I know it's late.

GFS is the west outlier now. Barring the EC Ens being that much west/ or if the roles were reversed would anyone believe the GFS if it were the east solution?

Cone narrowed in a huge way and what this run did do was STOP the EC movement all over the place which increases confidence the correct solution will be around the BM. Good news.

Link to comment
Share on other sites

Archived

This topic is now archived and is closed to further replies.

  • Recently Browsing   0 members

    • No registered users viewing this page.

×
×
  • Create New...